WBR0509

Revision as of 14:49, 25 September 2013 by Gonzalo Romero (talk | contribs) (Created page with "{{WBRQuestion |QuestionAuthor=Gonzalo A. Romero, M.D. [mailto:gromero@wikidoc.org] |ExamType=USMLE Step 1 |MainCategory=Pharmacology |SubCategory=Repro...")
(diff) ← Older revision | Latest revision (diff) | Newer revision → (diff)
Jump to navigation Jump to search
 
Author [[PageAuthor::Gonzalo A. Romero, M.D. [1]]]
Exam Type ExamType::USMLE Step 1
Main Category MainCategory::Pharmacology
Sub Category SubCategory::Reproductive
Prompt [[Prompt::A 61-year-old male comes to the clinic due to hair loss. He has a progressive history of baldness. He has tried many topical ointments and different products seen on TV, which have not been very helpful. He would like to look younger and to prevent losing further hair. The family doctor considers using Finasteride to treat his condition. Which of the following is true about the mechanism of action of this drug?]]
Answer A AnswerA::It is a non-steroidal competitive inhibitor of androgens at the testosterone receptor
Answer A Explanation AnswerAExp::
Answer B AnswerB::Inhibits the synthesis of steroid
Answer B Explanation AnswerBExp::
Answer C AnswerC::Inhibits the conversion of testosterone to DHT
Answer C Explanation AnswerCExp::
Answer D AnswerD::Inhibits steroid binding
Answer D Explanation AnswerDExp::
Answer E AnswerE::Binds to estrogen receptors
Answer E Explanation AnswerEExp::
Right Answer RightAnswer::
Explanation [[Explanation::

Educational Objective:
References: ]]

Approved Approved::No
Keyword WBRKeyword::Testosterone agonist, WBRKeyword::testosterone antagonist
Linked Question Linked::
Order in Linked Questions LinkedOrder::